Представление, при котором матрицы Паули преобразуются

В квантовой теории поля Пескина и Шредера существует тождество матриц Паули, которое связано с тождеством Фирца (уравнение 3.77).

(3,77) ( о мю ) α β ( о мю ) γ дельта "=" 2 ϵ α γ ϵ β дельта .
Автор объясняет, что

Можно понять тождество, заметив, что индексы α , γ преобразовать в представлении Лоренца Ψ л , пока β , дельта преобразовать в отдельное представление Ψ р , и вся величина должна быть инвариантом Лоренца.

Как можно увидеть α , γ и β , дельта преобразовать в другое представление?

Ответы (4)

Матрицы Паули — это инвариантные тензоры, связывающие левый и правый спиноры. Эти спиноры преобразуются в разные представления группы Лоренца (как вы упомянули) и, следовательно, обычно обозначаются разными индексами. Это легко увидеть в двухкомпонентной нотации, однако, если вы не знакомы с этой нотацией, это также можно увидеть в четырехкомпонентном лагранжиане:

ψ ¯ γ мю ψ "=" ψ γ 0 γ мю ψ "=" ( ψ р * ψ л * ) ( 0 о мю о ¯ мю 0 ) ( ψ л ψ р ) "=" ψ р * о мю ψ р + ψ л * о ¯ мю ψ л .
Затем можно показать, что ψ л * ( ψ р * ) превращается в правый (левый) спинор. Ясно о мю поле затем соединяет ψ р поле с ψ л поле. Мы можем записать эти сокращения более явно, обозначив индексы левостороннего представления греческими индексами и индексы правостороннего представления греческими индексами с точками:
ψ л α ˙ * ( о мю ) α α α ˙ ψ л α

Примечание: у вас может возникнуть соблазн подумать о ψ р и ψ л не как отдельные поля, а просто поля с действующими на них проекторами. Это делает всю эту тему очень запутанной, и я призываю к удобному мышлению с точки зрения двух компонентных полей как фундаментальных объектов, составляющих фермионы.

Спасибо за ответ. У меня такой же вопрос. Однако я не очень понял ваш ответ, не могли бы вы дать более конкретные пояснения с точки зрения простого языка? Я это понимаю ψ ¯ γ мю ψ вектор Лоренца, ψ является спиннером Лоренца, который преобразуется в соответствии с приводимым представлением группы (из-за некоторых требований физики). Когда вы показываете, что ψ ¯ γ мю ψ может быть записан как сумма двух членов, но это не обязательно (?) означает, что каждый член также является вектором Лоренца.
Итак, что означает " о мю поле соединяет правое поле Вейля с левым полем Вейля"? Другими словами, просто взглянув на один термин, как это сделали вы, в чем смысл утверждения, что α и α ˙ принадлежит другому представлению? Наконец, почему тождество Фирца может быть понято (выведено с точностью до константы) в терминах этих аргументов? Прошу прощения за мою неосведомленность и заранее большое спасибо!
Верно ли это первое уравнение? Я думаю, это должно быть ψ ¯ р о мю ψ р + ψ ¯ л о ¯ мю ψ л , поэтому я не могу найти, что означает "Clearly a о мю поле соединяет ψ ¯ р поле с ψ ¯ л поле".

У меня был тот же вопрос, и ссылка, предоставленная Qmechanic , кажется, основана на твердом понимании теории групп. Мне было интересно, можно ли просто понять преобразование индексов для этого конкретного вопроса только на основе учебника, используя минимальное количество знаний/аргументов из теории групп. Посоветовавшись с моим коллегой Альберто, вот ответ, который я получил по этому критерию.

Тождество (уравнение (3.77))) гласит:

( о мю ) α β ( о мю ) γ дельта "=" 2 ϵ α γ ϵ β дельта   .

Начнем с правой части равенства. Во-первых, мы можем показать, что антисимметричный символ ϵ α γ является лоренц-инвариантным, если оба индекса преобразуются как левые спиноры Вейля

Ψ л U л Ψ л "=" опыт ( я θ о 2 β о 2 ) Ψ л     .

Приведенное выше утверждение эквивалентно следующему тождеству

ϵ α γ опыт ( я θ о 2 β о 2 ) α α опыт ( я θ о 2 β о 2 ) γ γ ϵ α γ "=" ϵ α γ
или
опыт ( я θ о 2 β о 2 ) ( 0 1 1 0 ) опыт ( я θ о 2 β о 2 ) Т "=" ( 0 1 1 0 )

Вышеприведенное тождество можно без особого труда показать, заметив ( 0 1 1 0 ) "=" я о 2 , о "=" о и поперечное уравнение (3.38), так что

опыт ( я θ о 2 β о 2 ) о 2 опыт ( я θ о 2 β о 2 ) Т "=" опыт ( я θ о 2 β о 2 ) о 2 [ н 1 н ! ( я θ о 2 β о 2 ) н ] Т "=" опыт ( я θ о 2 β о 2 ) опыт ( + я θ о * 2 + β о * 2 ) Т о 2 "=" опыт ( я θ о 2 β о 2 ) опыт ( + я θ о 2 + β о 2 ) о 2 "=" опыт ( я θ о 2 β о 2 ) опыт ( + я θ о 2 + β о 2 ) о 2 "=" о 2

Очень похожее рассуждение показывает, что ϵ α γ также инвариантно, если оба индекса преобразуются как правые спиноры Вейля . Таким образом, можно рассматривать правую часть тождества как инвариантный тензор, где α , γ преобразует в представлении Лоренца Ψ л , пока β , дельта преобразует в отдельном представлении Лоренца Ψ р , как указано в учебнике.

Теперь переходим к левой части тождества. Математически более сложно (но все же осуществимо) показать (хотя умная догадка также убедительно указывает), что ( о мю ) α β также является лоренц-инвариантным тензором, когда мю преобразуется как вектор Лоренца, определяемый уравнением (3.19), α превращается в левый спинор и β превращается в правый спинор. Так что, когда мю свертывается в левой части тождества, остальные свободные параметры преобразуются точно так же, как и в правой части. Это сразу приводит к выводу, что тождество верно с точностью до постоянного числа, которое затем можно зафиксировать, оценивая только один член (вместо всех 2 4 "=" 16 из них).

Обычно приведенные выше аргументы приводятся на языке теории групп в более элегантной форме, и одной хорошей ссылкой является Квантовая теория поля Средненицкого (см. текст между уравнениями (34.18) и (35.20)).

Удобно использовать пунктирную запись. Алгебра Лоренца с о ( 1 , 3 ) изоморфен с ты ( 2 ) л × с ты ( 2 ) р . Обозначим фундаментальные с ты ( 2 ) л индексы по α , β и т.д. и фундаментальные с ты ( 2 ) р индексы по α ˙ , β ˙ и т. д. Заметим, что для унитарных представлений с о ( 1 , 3 ) , комплексное сопряжение меняет местами представления L и R и, следовательно, также меняет местами индексы без точек и с точками.

Есть несколько инвариантных по Лоренцу тензоров, представляющих интерес, в частности, дельта β α , дельта β ˙ α ˙ , ε α β , ε α ˙ β ˙ и матрицы Паули ( о мю ) α β ˙ Чтобы быть абсолютно ясным, я имею в виду, что когда они зажаты между спинорами, они трансформируются способом, указанным их индексами (см. ответ Джеффа на этот вопрос). Я оставляю вам в качестве домашнего задания доказательство того, что я только что сказал.

Если все индексы явные, то любое уравнение должно сохранять структуру индексов. Например, интересующее вас количество

( о мю ) α β ˙ ( о мю ) γ дельта ˙
Это инвариант Лоренца и имеет свободные индексы α , β ˙ , γ дельта ˙ . Единственная лоренц-инвариантная величина, которую можно построить из этих индексов, это ε α γ ε β ˙ дельта ˙ . Таким образом, мы должны иметь
( о мю ) α β ˙ ( о мю ) γ дельта ˙ "=" λ ε α γ ε β ˙ дельта ˙
Осталось исправить константу λ . Для этого можно установить β ˙ "=" γ и α "=" дельта ˙ и суммировать по повторяющимся индексам (это нарушает лоренц-инвариантность, но на данный момент нас это не волнует). Тогда мы имеем матричное уравнение
тр ( о мю о мю ) "=" λ тр ( ε 2 ) "=" тр ( я 2 ) + тр ( о о ) "=" λ тр ( я 2 ) λ "=" 2 .
Таким образом,
( о мю ) α β ˙ ( о мю ) γ дельта ˙ "=" 2 ε α γ ε β ˙ дельта ˙
Это соответствует вашему уравнению до знака, который зависит от соглашения. Я использовал соглашение о том, что ε 12 "=" ε 1 ˙ 2 ˙ "=" 1 .

@Pahar Единственное, чего я не понимаю, так это почему единственная возможная инвариантная величина Лоренца - это ε α γ ε β ˙ дельта ˙ ?
Между вашим доказательством и доказательством Пескина есть разница в знаке. Это связано с другим выбором метрик?
@SamapanBhadury - (1) что еще это может быть? Я уже записал для вас все лоренц-инвариантные величины. Остается построить что-то из них, но с правильной структурой индекса. (2) Да, это звучит правильно. мое метрическое соглашение (-+++)
почему нет дельта α β дельта γ дельта ? Почему мы не можем построить с таким дельта -функции?
@SamapanBhadury - дельта имеет один верхний индекс и один нижний индекс. У LHS снижены все четыре индекса.
ты хочешь сказать дельта β α не существует?
@SamapanBhadury Нет, нет. я имею в виду, что дельта β α является лоренц-инвариантным тензором, но дельта α β НЕ является, и нам разрешено использовать только лоренц-инвариантные тензоры, поскольку LHS является лоренц-инвариантным тензором.
не могли бы вы объяснить это утверждение или направить меня к ссылке.
@SamapanBhadury - см. комментарий выше.

Другой способ убедиться в этом — использовать законы преобразования для γ мю матрицы. Обратите внимание, что мы можем написать

Λ 1 / 2 "=" ( Λ 1 / 2 л 0 0 Λ 1 / 2 р ) ,
где Λ 1 / 2 это матрица, которая выполняет преобразование Лоренца на спиноре Дирака. Легко убедиться, что это так, потому что Λ 1 / 2 состоит из комбинаций единичной и блочно-диагональной матриц С мю ν (при условии, что мы используем киральное представление, как у Пескина и Шрёсдера). Легко видеть, что Λ 1 / 2 л - оператор, выполняющий преобразование Лоренца на левом спиноре, и Λ 1 / 2 р делает то же самое для правого спинора. Чтобы привести конкретный пример, для бесконечно малых преобразований уравнение 3.37 говорит, что
Λ 1 / 2 л "=" 1 я θ о / 2 β о / 2 , и  Λ 1 / 2 р "=" 1 я θ о / 2 + β о / 2.

Теперь мы можем использовать законы преобразования γ мю матрицы:

( Λ 1 / 2 л 1 0 0 Λ 1 / 2 р 1 ) ( 0 о мю о ¯ мю 0 ) ( Λ 1 / 2 л 0 0 Λ 1 / 2 р )
"=" Λ 1 / 2 1 γ мю Λ 1 / 2 "=" Λ мю ν γ ν "=" ( 0 Λ мю ν о ν Λ мю ν о ¯ ν 0 ) .
Выполняя умножение матриц и ориентируясь на правый верхний блок, получаем
Λ 1 / 2 л 1 о мю Λ 1 / 2 р "=" Λ мю ν о ν .